What is the inverse of the function f(x)
-x + 2?
q
O h(x) = 18x - 2
h(x) = 9x - 18
O h(x) = 9x + 18
® h(x) - 18x + 2

Answers

Answer 1

Answer:

[tex]h(x)= 9x - 18[/tex]

Step-by-step explanation:

Given

[tex]f(x) = \frac{1}{9}x + 2[/tex]

Required

The inverse

[tex]f(x) = \frac{1}{9}x + 2[/tex]

Replace f(x) with y

[tex]y = \frac{1}{9}x + 2[/tex]

Swap x and y

[tex]x = \frac{1}{9}y + 2[/tex]

Subtract 2

[tex]x - 2= \frac{1}{9}y[/tex]

Multiply 9

[tex]9x - 18 = y[/tex]

Rewrite as:

[tex]y = 9x - 18[/tex]

So:

[tex]h(x)= 9x - 18[/tex]


Related Questions

Which best describes the vertex of the graph?


a (-3, -4)
b (-3, -4)
c (3, -4)
d (3, -4)

Answers

Answer: C

Step-by-step explanation:

C. Is the answer for sure

Instructions: Problem 2 ! Find the missing angle in the image below. Do not include spaces in your answers

Answers

Step-by-step explanation:

since angles in a triangle add up to 180

<vuw=180-(71+23)

=86°

since angles in a straight line add up to 180

<vuf=180-86

=94

BRAINLIEST FOR ANSWER WITH EXPLANATION
JUST ONE QUESTION ATTACHED BELOW:

Answers

Answer:

m=16

Step-by-step explanation:

(6+6)/m=6/8

12/m=6/8

M=16

Answer:

16

Step-by-step explanation:

This triangle has been dialated; ILM is a dialated version of IJK. We know that dialations are by scale factors, and we also know the length of two corresponding sides.

IM is 6 m long, and IMK is 6+6 m long, so 12 m. So using simple ratios, we know the dialation factor from ILM to IJK is 2. The corresponding side for JK (m) is LM, which we know is 8 m. So side m is 16 because of the scale factor (2).

Maybe you thought we were solving for "m", as in a varriable all the sides are multiplied by, but I think that's just bad labeling. Hope this helps!

please tel me answer of under root 3+4i
without calculatot with steps

Answers

Hello,

[tex]Let's\ say \\\\z=\sqrt{3+4*i} =a+b*i\\\\z^2=3+4*i=(a+b*i)^2=a^2-b^2+2i*a*b\\\\\\if \ a\neq 0\\\left\{\begin{array}{ccc}a^2+b^2&=&3\\2ab=4\\\end{array}\right.\\\\\\\left\{\begin{array}{ccc}b=\dfrac{2}{a}\\a^2-(\dfrac{2}{a})^2=2\\\end{array}\right.\\\\\\a^4-4=3*a^2\\a^4-3a^2-4=0\\\\\Delta=(-3)^2-4*1*(-4)=25=5^2\\\\a^2=4\ or \ a^2=-1 (impossible)\\\\So:\\(a=2\ and\ b=1)\ or\ (a=-2\ and\ b=-1)\\[/tex]

Roots are thus 2+i and -2-i

There is an other using a geometrical formula (formule de Moivre)

a train leaves Westchester at 6:30. What time should it arrive at Middlewich

Answers

Answer:

a). 6:51

b). 6:30 am

c). 34 minutes

Step-by-step explanation:

a). Train leaves Westchester at 6:30.

   From the arrival - departure table in column (2),

   Arrival time of the train at middlewich = 6:51

b). Kate has to reach Southam before 9:00 am

   Therefore, time of the latest train that she can catch to get to work on time is 6:30 am

    By this train she can reach at 07:19 at Southam.

c). Duration of journey from Westchester to Eastwick = 06:34 - 06:00

                                                                                        = 00:34

                                                                                        ≈ 34 minutes

Find the measure of the missing angle using exterior angle sum theorm

Answers

35 degrees.

Assuming 145 is the exterior angle, the interior is 35. 180-145=35
Add 110 to 35 and get 145 degrees. Which leads the remaining angle 35 degrees

Answer:

35

Step-by-step explanation:

The exterior sum theorem states the exterior angle is equal to the sum of the opposite interior angles

145 = ?+110

145 - 100 = ?

35 = ?

Can someone help me?

Answers

Answer:

C

Step-by-step explanation:

its asking for y, on the graph, the line is placed on point 4

If the quadratic formula is used to find the solution set of 3x + 4x-2 = 0, what are the solutions? ​

Answers

Answer is c
hope it helps I’m new at this

helpppppppppppppppppp

Answers

Answer:

Dude

sheesh i only seeing 5 and 18
The answer is 5 it is the only number that can be multiplied by another number to get 10

Plis help me it’s for today

Answers

Answer:

Following are the solution to the given points:

Step-by-step explanation:

For question 1:

[tex]\to 3^{-4}= \frac{1}{3^4}=\frac{1}{81}=0.0123456789[/tex]

For question 2:

[tex]\to (-2)^{3}\cdot(-2)^{4}\cdot(-2)^{-1}=-8\cdot-16\cdot -\frac{1}{2}= 128\cdot -\frac{1}{2}=-64[/tex]

For question 3:

[tex]\to 7^{-4} \div 7^{-2}= \frac{1}{7^{4}} \div \frac{1}{7^{2}}=\frac{1}{7^{4}} \times \frac{7^{2}}{1}=\frac{1}{7^{2}} =\frac{1}{49} =0.0204081633[/tex]

For question 4:

[tex]\to [(-3)^{2}]^3= (-3)^{2\cdot 3}= (-3)^{6}=729[/tex]

For question 5:

[tex]\to [5 \cdot (-3)]^{2}= 25 \cdot 9=225[/tex]

For question 6:

[tex]\to [(10 \div 5)]^{3}= [(\frac{10}{5})]^{3}=[2]^{3}=8[/tex]

For question 7:

[tex]\to 10^6 \cdot 10^{-4} \cdot 10^2= 10^6 \cdot \frac{1}{10^{4}} \cdot 10^2= 10^2 \cdot 10^2=10^4=10,000[/tex]

For question 8:

[tex]\to (-4)^{-5}=\frac{1}{(-4)^{5}}=- \frac{1}{1,024}=-0.0009765625[/tex]

For question 9:

[tex]\to \frac{2^3}{2^4}= \frac{8}{16}=\frac{1}{2}=0.5[/tex]

For question 10:

[tex]\to (-6)^3 \cdot (-6)^5 \cdot (-6)^{-5}= (-6)^3 \cdot (-6)^5 \cdot \frac{1}{(-6)^{5}}= (-6)^3 =-216[/tex]

can someone answer plssss gives 100 pints i think bc I picked 100

Answers

[tex]1. \frac{20}{100} [/tex]

[tex]2. \frac{1}{5} [/tex]

Answer:  1

Explanation:

There are a few ways to do this. One way is to notice that the jump from 5 to 100 is "times 20" (go from right to left across the bottom denominators).

So we must do the same "times 20" type of jump when going across the numerators. If x is the numerator for the right hand side, then we go from x to 20. That must mean x = 1

Put another way, we could have these steps

20/100 = x/5

20*5 = 100*x ... cross multiplication

100 = 100x

100x = 100

x = 100/100 .... dividing both sides by 100

x = 1

We see that the fraction 20/100 reduces fully to 1/5

To go from 1/5 to 20/100, we multiply both parts by 20 (divide both parts by 20 to go in reverse).

if sam has 20 watermelons and he eats 14 how many does he have left?

Answers

Answer:

6 duh!

you can do this by yourself lol

Find the area of the triangle whose vertices are (2, 3); (-1, 0); (2, -4)...
[Also show the steps of the solution]
Please answer correctly, it's really urgent!​

Answers

Answer:

Area = 14.5

Step-by-step explanation:

Let's label the given coordinates A, B, C;

Thus;

A = (2, 3)

B = (-1, 0)

C = (2, -4)

From the coordinate geometry formula, the formula for area of a triangle with 3 vertices is;

Area = [Ax(By - Cy) + Bx(Cy - Ay) + Cx(Ay - By)]/2

Area = [2(0 - (-4)) + (-1)(-4 - 3) + 2(3 - (-4))]/2

Area = 29/2

Area = 14.5

Question 18 (5 points)
Determine the sum of the measures of the interior angles of a dodecagon (12-sided
polygon).
540°
1,800°
360°
2,160°

Answers

Answer:

its 540 bro

Step-by-step explanation:

In ΔCDE, the measure of ∠E=90°, ED = 28, CE = 45, and DC = 53. What ratio represents the tangent of ∠C?

Answers

Answer:

3/5

Step-by-step explanation:

prependicular / hypotenuse

sin C=ED/CD

=3/5

someone please help me ASAP!

Answers

Answer:

214°

Step-by-step explanation:

The measure of an arc that sees the center angle of the circle is equal to the very same angle that it sees

Since the measure of circle is 360° and arc AB is given as 146° the measure of arc ACB should be 360 - 146 = 214°

One number is 6 times a second number. The sum of the two numbers is 56. Find the numbersOne number is 6 times a second number.

The sum of the two numbers is 56. Find the numbers

Answers

Answer:

I think its 56 = 56

Step-by-step explanation:

Answer:

8 and 48

Step-by-step explanation:

This can be written as a system of equations. X will be the smaller number and y will be the larger:

6x = y

x + y = 56

Then substitute y in the second equation for 6x(seen in first equation) and solve.

x + 6x = 56

7x = 56

x = 8

So one number is 8. To find the other plug in 8 for x in either of the original equations (both get the same answer for y).

6x = y

6 * 8 = y

48 = y

Then double check by seeing if they fit the requirements of the problem.

Hope this helps!

Please help :(((((((((((((((((((((((((

Answers

jeez that looks really hard

Step-by-step explanation:

pls help me in these questions ​

Answers

Answer:

1=85

2=10

3=108

Step-by-step explanation:

Number 1: Calculate each angle...and you know a straight line is 180°. N the interior sum of a quadrilateral is always 360°.

Number2: Use corresponding, alternate and interior angles method. It will help

Number 3: It's just about solving the interior sum of the pentagon

Answer:

2. 10 degrees

3. 108 degrees

Step-by-step explanation:

sorry I reposted it because the previous answer was deleted and plz mark me as a brainliest.

Which graph represents the solution of x2 + 9y2 ≤ 81 and y2 + 2 < x? On a coordinate plane, an ellipse has center (0, 0) and goes through (3, 0), (0, negative 9), (negative 3, 0), and (0, 9). A parabola opens to the right and goes through (6, 2), has vertex (2, 0), and goes through (6, negative 2). Everything inside of the ellipse and outside of the parabola is shaded. On a coordinate plane, an ellipse has center (0, 0) and goes through (3, 0), (0, negative 9), (negative 3, 0), and (0, 9). A parabola opens to the right and goes through (6, 2), has vertex (2, 0), and goes through (6, negative 2). Everything inside of the ellipse and inside of the parabola is shaded. On a coordinate plane, an ellipse has center (0, 0) and goes through (9, 0), (0, negative 3), (negative 9, 0) and (0, 3). A parabola opens to the right and goes through (6, 2), has vertex (2, 0), and goes through (6, negative 2). Everything inside of the ellipse and inside of the parabola is shaded. On a coordinate plane, an ellipse has center (0, 0) and goes through (9, 0), (0, negative 3), (negative 9, 0) and (0, 3). A parabola opens to the right and goes through (6, 2), has vertex (2, 0), and goes through (6, negative 2). Everything inside of the ellipse and outside of the parabola is shaded.

Answers

9514 1404 393

Answer:

  C

Step-by-step explanation:

The ellipse y-intercepts are ±3, the x-intercepts are ±9, eliminating choices A and B. The parabola is shaded inside, eliminating choice D.

The correct choice is the third one.

The a oranial price of a skate broad was reduced by 15 dollars .the new price is 49 dollars if p=the stakebroads oranail price in dollars what mathematical sentence expresses the information
a.49-p=15
b.15-p=49
c.15+p=49
d.p-15=49

Answers

Answer:

d. p - 15 = 49

Step-by-step explanation:

p = original price

The price was reduced by 15 dollars.

The new price is p - 15.

The new price is 49.

p - 15 = 49

Answer: d. p - 15 = 49

I forgot to label the triangle below! I just know that the cos A = 0.48. Based on this information, which angle should be marked A?

Answers

Answer:

angle 1

Step-by-step explanation:

Using the trigonometric mnemonic SOH CAH TOA, we know that cos or cosine is the ratio between the adjacent side and hypotenuse side.

This means that if cos A = 0.48, A is the measure of the angle which it's relative adjacent side divided by the hypotenuse of the triangle will be around 0.48.

Let's try angle 2, cos (angle 2) = adjacent / hypotenuse = 7.8 / 8.9 = 0.876404494382 ≈ 0.87 ≠ 0.48. Since the proportions are not equal, this angle cannot be the one marked as A.

Since angle 3 is a right angle, the adjacent could be either side so it cannot be correct. Thus angle 1 is correct.

álgebra 1 solve -7 + 18(17h + 19)

Answers

Answer:  =360h+335

Step-by-step explanation:

Answer:

306h + 281

Step-by-step explanation:

-7 + 18(17h + 19)

-7 + 306h + 288

306h + 281

Solve for x

Marking brainliest

Answers

Answer:

110°

Step-by-step explanation:

30 points if helped, question in the picture

Answers

Step-by-step explanation:

[tex] \frac{2}{2 + \sqrt{3} } \times \frac{2 - \sqrt{3} }{2 - \sqrt{3} } = \frac{2(2 - \sqrt{3} )}{1} = 4 - 2 \sqrt{3} [/tex]

Hey Guys! Can You Please Help Me In My Math​

Answers

This would be a great tree diagram
Sorry...

pic is not clear.....

You invest $4.000 in a savings account. The account pays 3% annual interest. How much money will be in the savings account after 9 years?

A) $4,938.29
B) $5,219.20
C) $5,124.33
D) $6,003.45​

Answers

Answer:

4000*0.03*9=1080

4000+1080=5080

umm but like thats not one of the answers sorry

Hope This Helps!!!

Step-by-step explanation:

4000*3% = 120

120*9= 1080

1080+4000=5080

Surface area of a cuboid is 384
work out the volume

Answers

Answer:

is 512

Step-by-step explanation:

Ariel has 20 bouncy balls. He later then sold them for a price of $2 per ball to his best friend, Mike Hawk. Later, Ariel has gotten back 1/69 of his original amount of bouncy balls and got 20 more. How many bouncy balls does Ariel have?

Answers

ariel has 40 balls left

Answer:

420

Step-by-step explanation:

Write the equation of the graph y=

Answers

9514 1404 393

Answer:

  y = 6^x -3

Step-by-step explanation:

The graph is that of an exponential function that has been translated downward. We notice the horizontal asymptote is -3, and a couple of points on the graph are (0, -2) and (1, 3).

The shifted parent function will look like ...

  y = a·b^x +c

where c is the horizontal asymptote. Using the two points we found, we have ...

  -2 = a·b^0 -3 . . . . . using (x, y) = (0, -2)

  1 = a . . . . . . . . . . add 3 and simplify

Then using (x, y) = (1, 3), we have ...

  3 = b^1 -3

  6 = b . . . . . . . . . add 3 and simplify

So, the equation is ...

  y = 6^x -3

Other Questions
Helpppppppppppppppppppppppppppppppp Find the volume of the triangular prism."14 fth=3A6 ft What is the recursive formula for this geometric sequence?-3, -21, -147, -1029, ... Think about a suspense movie you've watched- one that makes you feel nervous about what might happen to the other characters American Industries' outstanding bonds have a 25-year maturity and $1,000 par value. Their nominal yield to maturity is 9.25%, they pay interest semiannually, and they sell at a price of $850. What is the bond's nominal coupon interest rate Sophia is putting together a training manual for her new batch of employees.Which of the following features can she use to add a document title at the top of each page? A) Heading style B) Page margin C) References D) Header and Footer Drag the tiles to the correct boxes to complete the pairs.Match each scenario to the climatic factor that caused it. How can I pass the variable argument list passed to one function to another function. Question 3 of 24Suppose a novel tells about a young girl growing up poor in a rural town. Sheovercomes hardship and obstacles, including the small-minded expectationsof her society, to better her situation through hard work and ingenuity. Whichof these would best symbolize her struggle?A. A feral cat with only one eye that manages to keep stealingenough food to stay aliveB. A recycled glass wind chime that signals changes in the weatherand approaching stormsC. An image of an eye that she seems to encounter in various placesat key momentsD. A movement taking place in the nearby town to install the firstelectric stoplight Compare 3/10 and 1/5 by creating common denominators. then draw fractions models to show that you have written the correct sign. PELASEEEEEE Ocean life appears to be affected by a relatively small number of variables such as temperature, density, available sunlight, available nutrients. If any one of these variables changes how do you think it might it impact the human food web? the major attractions of of province 1 15. A line is given by the equation y = 49. What is the equationof the perpendicular line that passes through the point (12, 35)?The equation of the line is given by x Determine another point on the parabola that has an axis of symmetry x = 4 and a point on the parabola is (0, 2), Another point on the parabola is An emergency relief plane is dropping a care package from a plane to a group of medical personnel working for a relief agency in an African village. The package is designed to land in a small lake, inflate an attached raft upon impact, and finally resurface with the raft side down. The plane will be moving horizontally with a ground speed of 59.1 m/s. The package will be dropped a horizontal distance of 521 m from the intended target location. At what altitude above the pond must the plane be flying in order to successfully accomplish this feat? Need help on this!!! 6 points!!! During an experiment, solid iodine was placed in a sealed container. The container was gradually heated and purple-colored vapors of iodine formed were observed. Describe this system when it reaches phase equilibrium. (10 points) A. Directions:Describe ways on how to protect humans and plants and animals Choose your answer in the box and fill in the concept map below 26Which defines a circle? You invest $15,000 into two different accounts. One of the accounts has 4% interest and the other has 3.2% interest. After one year, you have accumulated a total of $545.60 in interest. How much was initially invested in the account with 4% interest?